Download as pdf or txt
Download as pdf or txt
You are on page 1of 4

Dominated convergence theorem

In measure theory, Lebesgue's dominated convergence theorem provides sufficient conditions under which almost
everywhere convergence of a sequence of functions implies convergence in the L1 norm. Its power and utility are two of the
primary theoretical advantages of Lebesgue integration over Riemann integration.

In addition to its frequent appearance in mathematical analysis and partial differential equations, it is widely used in probability
theory, since it gives a sufficient condition for the convergence of expected values of random variables.

Statement
Lebesgue's dominated convergence theorem.[1] Let be a sequence of complex-valued measurable functions on a
measure space . Suppose that the sequence converges pointwise to a function and is dominated by some integrable
function in the sense that

for all numbers n in the index set of the sequence and all points . Then f is integrable (in the Lebesgue sense) and

which also implies

Remark 1. The statement "g is integrable" means that measurable function is Lebesgue integrable; i.e.

Remark 2. The convergence of the sequence and domination by can be relaxed to hold only μ-almost everywhere provided
the measure space (S, Σ, μ) is complete or is chosen as a measurable function which agrees μ-almost everywhere with the
μ-almost everywhere existing pointwise limit. (These precautions are necessary, because otherwise there might exist a non-
measurable subset of a μ-null set N ∈ Σ, hence might not be measurable.)

Remark 3. If , the condition that there is a dominating integrable function can be relaxed to uniform integrability
of the sequence (fn ), see Vitali convergence theorem.

Remark 4. While is Lebesgue integrable, it is not in general Riemann integrable. For example, take fn to be defined in
so that it is one at rational numbers and zero everywhere else (on the irrationals). The series (fn ) converges pointwise to
0, so f is identically zero, but is not Riemann integrable, since its image in every finite interval is and
thus the upper and lower Darboux integrals are 1 and 0, respectively.

Proof
Without loss of generality, one can assume that f is real, because one can split f into its real and imaginary parts (remember that
a sequence of complex numbers converges if and only if both its real and imaginary counterparts converge) and apply the
triangle inequality at the end.

Lebesgue's dominated convergence theorem is a special case of the Fatou–Lebesgue theorem. Below, however, is a direct
proof that uses Fatou’s lemma as the essential tool.

Since f is the pointwise limit of the sequence (fn ) of measurable functions that are dominated by g, it is also measurable and
dominated by g, hence it is integrable. Furthermore, (these will be needed later),
for all n and

The second of these is trivially true (by the very definition of f). Using linearity and monotonicity of the Lebesgue integral,

By the reverse Fatou lemma (it is here that we use the fact that |f−fn | is bounded above by an integrable function)

which implies that the limit exists and vanishes i.e.

Finally, since

we have that

The theorem now follows.

If the assumptions hold only μ-almost everywhere, then there exists a μ-null set N ∈ Σ such that the functions fn 1S \ N satisfy
the assumptions everywhere on S. Then the function f(x) defined as the pointwise limit of fn (x) for x ∈ S \ N and by f(x) = 0 for
x ∈ N, is measurable and is the pointwise limit of this modified function sequence. The values of these integrals are not
influenced by these changes to the integrands on this μ-null set N, so the theorem continues to hold.

DCT holds even if fn converges to f in measure (finite measure) and the dominating function is non-negative almost
everywhere.

Discussion of the assumptions


The assumption that the sequence is dominated by some integrable g cannot be dispensed with. This may be seen as follows:
define fn (x) = n for x in the interval (0, 1/n] and fn (x) = 0 otherwise. Any g which dominates the sequence must also dominate
the pointwise supremum h = supn fn . Observe that

by the divergence of the harmonic series. Hence, the monotonicity of the Lebesgue integral tells us that there exists no
integrable function which dominates the sequence on [0,1]. A direct calculation shows that integration and pointwise limit do
not commute for this sequence:
because the pointwise limit of the sequence is the zero function. Note that the sequence (fn ) is not even uniformly integrable,
hence also the Vitali convergence theorem is not applicable.

Bounded convergence theorem


One corollary to the dominated convergence theorem is the bounded convergence theorem, which states that if (fn ) is a
sequence of uniformly bounded complex-valued measurable functions which converges pointwise on a bounded measure
space (S, Σ, μ) (i.e. one in which μ(S) is finite) to a function f, then the limit f is an integrable function and

Remark: The pointwise convergence and uniform boundedness of the sequence can be relaxed to hold only μ-almost
everywhere, provided the measure space (S, Σ, μ) is complete or f is chosen as a measurable function which agrees μ-almost
everywhere with the μ-almost everywhere existing pointwise limit.

Proof

Since the sequence is uniformly bounded, there is a real number M such that |fn (x)| ≤ M for all x ∈ S and for all n. Define
g(x) = M for all x ∈ S. Then the sequence is dominated by g. Furthermore, g is integrable since it is a constant function on a
set of finite measure. Therefore, the result follows from the dominated convergence theorem.

If the assumptions hold only μ-almost everywhere, then there exists a μ-null set N ∈ Σ such that the functions fn 1S\N satisfy
the assumptions everywhere on S.

Dominated convergence in Lp-spaces (corollary)


Let be a measure space, a real number and a sequence of -measurable functions
.

Assume the sequence converges -almost everywhere to an -measurable function , and is dominated by a
(cf. Lp space), i.e., for every natural number we have: , μ-almost everywhere.

Then all as well as are in and the sequence converges to in the sense of , i.e.:

Idea of the proof: Apply the original theorem to the function sequence with the dominating function .

Extensions
The dominated convergence theorem applies also to measurable functions with values in a Banach space, with the dominating
function still being non-negative and integrable as above. The assumption of convergence almost everywhere can be
weakened to require only convergence in measure.

The dominated convergence theorem applies also to conditional expectations. [2]

See also
Convergence of random variables, Convergence in mean
Monotone convergence theorem (does not require domination by an integrable function but assumes
monotonicity of the sequence instead)
Scheffé's lemma
Uniform integrability
Vitali convergence theorem (a generalization of Lebesgue's dominated convergence theorem)

Notes
1. For the real case, see Evans, Lawrence C; Gariepy, Ronald F (2015). Measure Theory and Fine Properties of
Functions. CRC Press. pp. Theorem 1.19.
2. Zitkovic 2013, Proposition 10.5.

References
Bartle, R.G. (1995). The Elements of Integration and Lebesgue Measure (https://books.google.com/books?id
=8oNGAAAAYAAJ). Wiley Interscience. ISBN 9780471042228.
Royden, H.L. (1988). Real Analysis (https://books.google.com/books?id=J4k_AQAAIAAJ). Prentice Hall.
ISBN 9780024041517.
Weir, Alan J. (1973). "The Convergence Theorems". Lebesgue Integration and Measure. Cambridge:
Cambridge University Press. pp. 93–118. ISBN 0-521-08728-7.
Williams, D. (1991). Probability with martingales. Cambridge University Press. ISBN 0-521-40605-6.
Zitkovic, Gordan (Fall 2013). "Lecture10: Conditional Expectation" (https://web.ma.utexas.edu/users/gordanz/
notes/conditional_expectation.pdf) (PDF). Retrieved December 25, 2020.

Retrieved from "https://en.wikipedia.org/w/index.php?title=Dominated_convergence_theorem&oldid=1161163295"

You might also like